Đến nội dung

baopbc nội dung

Có 386 mục bởi baopbc (Tìm giới hạn từ 25-05-2020)



Sắp theo                Sắp xếp  

#613687 Về bài hình học trong cuộc thi Sharygin năm 2015

Đã gửi bởi baopbc on 08-02-2016 - 21:46 trong Tài liệu, chuyên đề, phương pháp về Hình học

Tác giả xin được dành lời cảm ơn chân thành tới bạn Nguyễn Tiến Dũng sinh viên K50 Đại học ngoại thương đã đóng góp nhiều lời giải cho tác giả đồng thời giúp tác giả đọc lại và hoàn thiện bài viết này.

 

Link http://analgeomatica...i-sharygin.html

 

Chúc mừng năm mới Bính Thân mọi điều như ý!

Sao em không vào được trang thầy ạ!

P/s: Thời gian này trang của thầy rất khó vào!




#623077 Việt Nam TST 2016 - Thảo luận đề thi

Đã gửi bởi baopbc on 27-03-2016 - 21:52 trong Thi HSG Quốc gia và Quốc tế

Lời giải và bình luận của mình cho hai bài hình thi TST năm 2016

 

http://analgeomatica...-toan-hinh.html

Thầy có thể trích dẫn file trực tiếp được không ạ! Một số thành viên có thể không đọc được vì sử dụng mạng $VNPT$!

Như em bây giờ chẳng hạn! :)




#628674 USAMO 2016

Đã gửi bởi baopbc on 21-04-2016 - 11:55 trong Thi HSG Quốc gia và Quốc tế

Bài 1. Cho $\mathbb{X}_{1}, \mathbb{X}_{2}, \cdots , \mathbb{X}_{100}$ là các tập con khác rỗng đôi một khác nhau của tập $\mathbb{S}$. Hai tập $\mathbb{X}_{i}$ và $\mathbb{X}_{i + 1}$ bất kỳ thì giao của chúng là bằng rỗng và hợp của chúng không là cả một tập $\mathbb{S}$, nói cách khác, $\mathbb{X}_{i} \cap \mathbb{X}_{i + 1} = \varnothing$ và $\mathbb{X}_{i} \cup \mathbb{X}_{i + 1} \neq \mathbb{S}$, với mọi $i \in \{1, \cdots , 99\}$. Tìm số phần tử nhỏ nhất có thể của $\mathbb{S}$

Lời giải bài $1$ của em:

Do $S$ có nhiều hơn $100$ tập con khác nhau (tính cả rỗng) nên $\left | S \right |\geq log_2 100$

$\Rightarrow \left | S \right |\geq 7$

Giả sử $\left | S \right |_{min}= 7$. Ta có nhận xét sau:

Với mỗi phần tử $X_i$ thỏa mãn yêu cầu bài toán thì có tối đa $5$ phần tử

Với mỗi phần tử $X_i$ thỏa mãn yêu cầu bài toán và có $5$ phần tử thì $X_{i+1}$ phải có $1$ phần tử

Với mỗi phần tử $X_i$ thỏa mãn yêu cầu bài toán và có $4$ phần tử thì $X_{i+1}$ phải có $1$ hoặc $2$ phần tử

$\Rightarrow $ số phần tử tối đa thỏa mãn yêu cầu bài toán là $C_7^1+C_7^2+C_7^3+C_7^1+C_7^2+C_7^1=7+21+35+7+21+7=98$ (phần tử) (Mâu thuẫn)

$\Rightarrow \left | S \right |\geq 8$

Với $\left | S \right |= 8$. Ta sẽ chỉ ra một tập các số thỏa mãn.

Em chỉnh lại như sau: Ý tưởng vẫn như cũ nhưng thay vì cố định tập $T$, ta có thể thay đổi và chuyển về dưới dạng các tập con của $S$.

Trước tiên ta xét các tập $X_i$ có $1$ phần tử. Xét $S=\left \{ 1;2;3;4;5;6;7;8 \right \}$. Ta viết các tập con của $S$ có một phần tử như sau:

$\left \{ 1 \right \}...\left \{ 2 \right \}...\left \{ 3 \right \}...\left \{ 4 \right \}...\left \{ 5 \right \}...\left \{ 6 \right \}...\left \{ 7 \right \}...\left \{ 8 \right \}$

Theo ý tưởng như trên ta sẽ chèn vào vị trí giữa $X_1$ và $X_3$ tập $X_2$ sao cho $X_2=T\X_1\cup X_2$

Tức là: $\left \{ 1 \right \}\left \{ 3;4;5;6;7;8 \right \}\left \{ 2 \right \}$. Tương tự với các tập khác.

Trong trường hợp $X_i$ có hai phần tử, ta sẽ bắt đầu từ $\left \{ 1;2 \right \}...\left \{ 2;3 \right \}$ và kết thúc bằng $\left \{ 7;8 \right \}$

Do hợp của các phần tử ở đây là đôi một khác nhau nên các tập ở giữa các tập có hai phần tử trên là đôi một khác nhau.

Thực hiện tương tự với $X_i$ có nhiều hơn $2$ phần tử ta thu được các tập $X$ cần tìm. $\blacksquare$.




#643652 USA TSTST 2016

Đã gửi bởi baopbc on 04-07-2016 - 19:14 trong Thi HSG Quốc gia và Quốc tế

Bài 6. Bổ đề. Cho tam giác $ABC$. Tiếp tuyến tại $A$ của $(ABC)$ cắt đường thẳng qua $B\perp AB$, đường thẳng qua $C\perp AC$ tại $P,Q.M\equiv PC\cap BQ.A'$ là đối xứng của $A$ qua trung điểm $BC$. Khi đó $MA'$ đi qua chân đường cao hạ từ $A$ xuống $BC$.

Chứng minh. $H$ là hình chiếu của $A$ lên $BC.N\equiv BP\cap CQ$

Dễ thấy $A'$ là trực tâm tam giác $BNC$ nên $A'$ thuộc trục đẳng phương của $(BQ),(PC)$. Do $P,B,C,Q$ đồng viên nên $MP.MC=MQ.MB$ suy ra $M$ nằm trên trục đẳng phương của $(BQ),(QC)$. Mặt khác dễ thấy $\frac{HB}{HC}=\frac{AP}{AQ}=\frac{HC}{HY}$ nên $HB.HY=HC.HX$ suy ra $H$ thuộc trục đẳng phương của $(BQ)(CP)$

Do đó $M,H,A'$ thẳng hàng.$\blacksquare$

Post 246.png

Giải bài toán:

$M,N$ theo thứ tự là trung điểm $DE,DF.\left \{ A_1;A_2 \right \}\equiv MN\cap (I)$ Do $NA_1.NA_2=NF.ND=ND^2=NB.NI$ nên $B,A_1,C,I$ đồng viên. Tương tự thì $C,A_2,B,I$ đồng viên do đó $\left \{ A_1;A_2 \right \}\equiv (BIC) \cap (I)$. Tương tự thì $B_1B_2,C_1C_2$ là các đường trung bình của tam giác $DEF$.

Post 247.png

$P$ là trung điểm $EF$ suy ra $P\equiv B_1B_2\cap C_1C_2$. Do $PB_1.PB_2=PC_1.PC_2$ nên $P$ thuộc trục đẳng phương của $(BB_1B_2)$ và $(CC_1C_2)$. Mặt khác do $MB_1.MB_2=MA_1.MA_2$ nên $M$ thuộc trục đẳng phương của $(BIC)$ và $(BB_1B_2)$. Tương tự thì $N$ thuộc trục đẳng phương của $(BIC)$ và $(CC_1C_2)$ do đó theo định lí về tâm đẳng phương thì trục đẳng phương của $(BB_1B_2)$ và $(CC_1C_2)$ đi qua giao điểm của $BM,CN$.

 

$X\equiv BM\cap CN$ suy ra $PX$ là trục đẳng phương của $(BB_1B_2)$ và $(CC_1C_2)$. Theo bổ đề suy ra $PX$ đi qua hình chiếu của $D$ lên $MN$ do đó $PX$ chia đôi $DK.\blacksquare$




#643664 USA TSTST 2016

Đã gửi bởi baopbc on 04-07-2016 - 20:44 trong Thi HSG Quốc gia và Quốc tế

Bài 2.

Post 248.png

 Do $OM\perp AG$ nên $PA$ là tiếp tuyến tại $A$ của $(AH)$ do đó $AG$ là đường đối cực $P$ đối với $(AH)$.

Gọi $E,F$ theo thứ tự là hình chiếu của $B,C$ lên $CA,AB$. Theo định lí về tâm đẳng phương thì $EF,AG,BC$ đồng quy tại $T$. 

Dễ thấy $EF$ là đường đối cực của $N$ đối với $(AH)$ do đó $PN$ là đường đối cực của $T$ đối với $(AH)$ suy ra $MT\perp NP$.

 

$D$ là hình chiếu của $A$ lên $BC$. Do $MXDN$ nội tiếp nên $TX.TM=TD.TN$. Mặt khác do $(TD,BC)=-1$ nên theo hệ thức $Maclaurin$ suy ra $TB.TC=TD.TN$ do đó $TX.TM=TB.TX$ kéo theo $MXBC$ nội tiếp. Từ đây chú ý $D$ là trung điểm $TN$ nên $X$ thuộc $(GQN)$ do đó ta có điều phải chứng minh.$\blacksquare$

 

PS.




#651677 Tuần 5 tháng 8/2016: Bài toán chia đôi đoạn thẳng

Đã gửi bởi baopbc on 28-08-2016 - 19:03 trong Chuyên mục Mỗi tuần một bài toán Hình học

Lời giải của em. Tính chất sau quen thuộc : Gọi $X,Y$ lần lượt là điểm chính giữa cung $AC$ không $B$ và cung $AB$ không chứa $C$.

Khi đó ta có $IE\cdot BX=IF\cdot CY$.

Gọi $S$ là giao điểm các tiếp tuyến tại $B,C$ của $(O)$ thì $Z$ là điểm chính giữa cung $BC$ không chứa $O$ của $(K)$.

Đường thẳng qua $Z$ song song với $EF$ cắt $CF,BE$ tại $T,S$. Chú ý các cặp góc $\angle ZCF=\angle YXC,\angle ZCE=\angle BYX$ và $ZB=ZC$ dễ thấy $Z$ là trung điểm $ST$.Do đó mà $IZ$ chia đôi $EF$. Mặt khác theo định lí Protassov's thì $PI$ là phân giác $\angle BPC$ nên $PI$ đi qua điểm chính giữa cung $BC$ không chứa $O$ của $(K)$ (tức $Z$). Do đó $PI$ chia đôi $EF.\ \blacksquare$

Post 305.PNG

Hình vẽ bài toán

PS. Định lí Protassov's là một kết quả quen thuộc và đã từng xuất hiện trên tạp chí THTT và trong bài viết của GGTH 2013.

Có thể tham khảo bài chứng minh sau của Jean Louis.




#651670 Tuần 5 tháng 8/2016: Bài toán chia đôi đoạn thẳng

Đã gửi bởi baopbc on 28-08-2016 - 18:32 trong Chuyên mục Mỗi tuần một bài toán Hình học

Như vậy thầy Hùng đã đưa ra lời giải bài cũ trong tuần 5 tháng 8 và kèm theo đó là bài toán mới, xin được trích dẫn lại bài toán mới.

 

Cho tam giác $ABC$ nhọn có phân giác $BE,CF$ cắt nhau tại $I$. Đường tròn $(K)$ đi qua $B,C$ và tâm ngoại tiếp $O$ của tam giác $ABC$. Đường tròn $(L)$ nằm trong tam giác tiếp xúc $CA,AB$ và tiếp xúc ngoài $(K)$ tại $P$. Chứng minh rằng $PI$ chia đôi $EF$.

Post 304.PNG

Hình vẽ bài toán




#642433 Tuần 5 tháng 6/ 2016 - Tổng quát ELMO 2016

Đã gửi bởi baopbc on 27-06-2016 - 14:04 trong Chuyên mục Mỗi tuần một bài toán Hình học

Một bài toán khá thú vị xung quanh cấu hình của bài toán gốc! :)

 

Bài toán. Cho tam giác $ABC$, tâm nội tiếp $I.(I)$ tiếp xúc $BC,CA,AB$ theo thứ tự tại $D,E,F$. Phân giác $\angle BAC$ cắt $DE,DF$ theo thứ tự tại $M,N$. Đường tròn $(MN)$ cắt $BC$ tại $P,Q$.

a, Chứng minh rằng $(APQ)$ tiếp xúc $(I)$ tại $R$.

b, $F_eR$ cắt $DE,DF$ theo thứ tự tại $M',N'$. Đường tròn $(M'N')$ cắt $BC$ tại $P',Q'$. Chứng minh rằng trục đẳng phương của $(AP'Q')$ và $(I)$ chia đôi $BC$

Post 234.png

Lời giải bài toán có thể tham khảo lời giải của yetti tại đây.




#642364 Tuần 5 tháng 6/ 2016 - Tổng quát ELMO 2016

Đã gửi bởi baopbc on 26-06-2016 - 22:37 trong Chuyên mục Mỗi tuần một bài toán Hình học

Lời giải. Để ý $D(BA,FA)=-1\Longleftrightarrow D(PA,NM)=-1$ mặt khác chú ý $\angle MPN=90^\circ$ nên $PM$ là phân giác $\angle APQ$. Tương tự thì $QM$ là phân giác $\angle AQP$

$\implies M$ là tâm nội tiếp tam giác $APQ$. Mặt khác để ý $DE$ đi qua $X$ và $(I)$ tiếp xúc $CB,CA$ tại $D,E$ nên theo bổ đề Sayawama dạng đảo thì $(I)$ tiếp xúc $(APQ).\blacksquare$




#642363 Tuần 5 tháng 6/ 2016 - Tổng quát ELMO 2016

Đã gửi bởi baopbc on 26-06-2016 - 22:34 trong Chuyên mục Mỗi tuần một bài toán Hình học

Như vậy thầy Hùng đã đưa ra lời giải bài cũ trong tuần 5 tháng 6 và kèm theo đó là bài toán mới, xin trích dẫn lại bài toán mới

 

Cho tam giác $ABC$ có đường tròn nội tiếp $(I)$ tiếp xúc $BC,CA,AB$ tại $D,E,F$. Một đường thẳng đi qua $A$ cắt $DE,DF$ tại $M,N$ sao cho đường tròn đường kính $MN$ cắt đoạn $BC$ tại $P,Q$. Chứng minh rằng đường tròn ngoại tiếp tam giác $APQ$ tiếp xúc $(I)$

Post 233.png




#623165 Tuần 5 tháng 3/2016: Một mở rộng của bài thi Vietnam TST 2016

Đã gửi bởi baopbc on 28-03-2016 - 16:14 trong Chuyên mục Mỗi tuần một bài toán Hình học

Thầy có thể giải thích rõ chỗ $\triangle BAU\sim \triangle CAV$ rồi suy ra điều phải chứng minh như thế nào không ạ! Em hơi thắc mắc chỗ này! Với lại ở đó hai tam giác $BAU, CAV$ không phải tam giác vuông thầy ạ! Nếu nó vuông thì hóa ra $B,K,U; C,L,V$ thẳng hàng rồi!  :mellow: Tuy nhiên hai tam giác đó đồng dạng thì không vấn đề gì! :)




#623161 Tuần 5 tháng 3/2016: Một mở rộng của bài thi Vietnam TST 2016

Đã gửi bởi baopbc on 28-03-2016 - 15:59 trong Chuyên mục Mỗi tuần một bài toán Hình học

Không biết mọi người hiểu như thế nào nhưng chỗ đó em đã nhấn mạnh chữ đoạn rất kĩ vì đây là chi tiết quan trọng! Phép đồng dạng này hồi cấp 2 em đã thắc mắc rất nhiều và sau mấy năm em vẫn chưa tìm được lời giải thích chính đáng! Em lí luận như sau:

Trước tiên ta có $E,F$ thuộc đoạn $BC$.

$\Rightarrow \angle KAM, \angle LAN$ tù.

Ta lấy điểm $M$ bằng cách vẽ đường tròn tâm $K$, bán kính $KB$. Khi đó tồn tại một điểm $M'$ thuộc đường thẳng $AC$ khác $M$ sao cho $KM'=KM=KB$ 

Giả sử $M'$ thuộc đoạn $AC$ thì $\angle KM'M > \angle KAM$

$\Rightarrow \angle KM'M$ tù. Mặt khác $\angle AM'M$ là góc ở đáy của tam giác cân nên điều này không thể xảy ra! Vậy tồn tại duy nhất điểm $M$ thuộc đoạn $AC$ sao cho $KM=KB$.

Tương tự thì tồn tại duy nhất điểm $N$ thuộc đoạn $AB$ sao cho $LN=LC$. Vậy nên để đảm bảo tính duy nhất thì hai tam giác này phải đồng dạng!

Chỗ này chứng minh như sau:

Post 27.png

Thân ái!




#623116 Tuần 5 tháng 3/2016: Một mở rộng của bài thi Vietnam TST 2016

Đã gửi bởi baopbc on 28-03-2016 - 00:46 trong Chuyên mục Mỗi tuần một bài toán Hình học

Lời giải bài cũ, tuần 5 tháng 3.

 

Bài 32. Cho tam giác $ABC$ với góc $A$ tù và đường cao $AH$. Các điểm $E,F$ thuộc đoạn $BC$ sao cho $\angle EAB=\angle FAC$. Gọi $P,Q$ lần lượt là đối xứng của $E,F$ qua $H$. Lấy $K$ trên trung trực $BP$ sao cho $AK\perp AF$. Lấy $L$ trên trung trực $CQ$ sao cho $AL\perp AE$. Lấy $M,N$ lần lượt thuộc đoạn $CA,AB$ sao cho $KM=KP$ và $LN=LQ$. Chứng minh rằng bốn điểm $B,C,M,N$ cùng thuộc một đường tròn.

Bài toán được thầy mở rộng từ bài $Vietnam TST 2016$ ngày 2 ý b. Mọi người có thể xem tại đây:

 https://www.dropbox....ay1295.pdf?dl=0

Lời giải. Trước tiên ta chứng minh: $\frac {AK}{AL}=\frac {AB}{AC}(*)$

Thật vậy gọi $Y, Z$ lần lượt là hình chiếu của $K, L$ lên $BC$ ta có: $AK.AH=HY.AF=BE.AF/2, AL.AH=HZ.AE=CF.AE/2$

$\Rightarrow \frac {AK}{AL}=\frac {BE.AF}{CF.AE}$

$(*) \Leftrightarrow \frac {BE.AF}{CF.AE}=\frac {AB}{AC}$ (đúng do $\angle EAB=\angle FAC$)

Vậy $\frac {AK}{AL}=\frac {AB}{AC}$

Post 25.png

Mặt khác do $AE,AF$ đẳng giác trong $\angle A$ nên $\angle KAB=\angle LAC$

$\Rightarrow \triangle KAB\sim \triangle LAC(c.g.c)$

Do $M,N$ thuộc đoạn $AB,AC; E, F$ thuộc đoạn $BC$ nên ta suy ra

$\triangle KNA\sim \triangle LMA(c.g.c)$

$\Rightarrow \frac {AN}{AM}=\frac {AK}{AL}=\frac {AB}{AC}$

$\Rightarrow B,M,N,C$ đồng viên. $\blacksquare$

Post 26.png

P/s:




#663427 Tuần 5 tháng 11/2016 : Mở rộng bài toán hình học trường đông tại Vinh năm 2016

Đã gửi bởi baopbc on 29-11-2016 - 23:01 trong Chuyên mục Mỗi tuần một bài toán Hình học

Giải. Ta có $\angle BEA+\angle CFA=\angle BPA+\angle CPA=180^\circ$ nên $BE$, $CF$ cắt nhau trên đường tròn $(AEF)$.

Gọi $H$ là trực tâm tam giác $ABC$. $R$ là giao điểm của $BE$ và $CF$.

Do $\angle BRC=\angle BHC=180^\circ-\angle BAC$ nên tứ giác $BHRC$ nội tiếp suy ra $R$ thuộc đường tròn đối xứng với đường tròn $(O)$ qua $BC$.

Gọi $X$ là trung điểm $BC$. $R_1$, $R_2$ là các giao điểm của $XR$ với đường tròn $(O)$. $A_1$, $A_2$ là giao điểm của $AX$ với các đường tròn $(BRC)$ và $(O)$.

Do tính đối xứng nên $\overline{XR}\cdot \overline{XR_1}=-\overline{XR_2}\cdot \overline{XR_1}=-\overline{XA_1}\cdot \overline{XA}=\overline{XA_2}\cdot \overline{XA}$ $\Rightarrow $ tứ giác $AR_1RA_2$ nội tiếp.

Mặt khác do $\overline{XR}=-\overline{XR_2}$ nên tứ giác $RCR_2B$ là hình bình hành $\Rightarrow $ $CR_2\parallel ER$.

Theo định lí Reim, tứ giác $AR_1RE$ nội tiếp. Tương tự ta thu được lục giác $AR_1FRA_2E$ nội tiếp. $\qquad (1)$

Từ $(1)$ $\Rightarrow$ $K$ thuộc trung trực $AA_2$. Mặt khác khi $P$ trùng chân đường vuông góc kẻ từ $A$ xuống $BC$ thì $K$ trùng trung điểm $AH$.

Do đó $K$ thuộc đường thẳng qua trung điểm $AH$ và vuông góc với $AX$. $\qquad (2)$

Gọi $K_1$ là trung điểm $AH$. $\triangle D_1E_1F_1$ là tam giác pedal của trực tâm $H$ đối với $\triangle ABC$.

Do $K_1X\perp E_1F_1$ và $E_1F_1\parallel AQ$ nên $K_1X\perp AQ$ $\Rightarrow $ $\triangle K_1D_1X\sim \triangle QD_1A$ (góc - góc)

$\Rightarrow $ $\tfrac{D_1A}{D_1Q}=\tfrac{D_1X}{D_1K_1}$ $\Rightarrow $ $\tfrac{D_1X}{D_1A}=\tfrac{D_1K_1}{D_1Q_1}$ $\Rightarrow $ $\triangle K_1D_1Q\sim \triangle XD_1A$ (cạnh - góc - cạnh).

$\implies QK_1\perp AX$ $\qquad (3)$

Từ $(2)$ và $(3)$ $\implies K$ thuộc $QK_1$ hay $ST\equiv QK_1$. $\qquad (4)$

Gọi $K_2$ là giao điểm của $SF$ và $TE$ thì $K_2$ là điểm đối xứng của $A$ qua $K$ suy ra $HK_2\parallel ST$.

Gọi $S_1$, $T_1$ lần lượt là giao điểm của $CH$, $BH$ với $ST$. Theo tính chất hình bình hành $\triangle K_2ST=\triangle HS_1T_1$ $\qquad (5)$

Đường thẳng qua $B$ vuông góc với $BA$ cắt $AS_1$ tại $M_1$. Tương tự xác định $N_1$.

Dễ thấy $\angle S_1HK_1=\angle ABX$, $\angle K_1S_1H=\angle BAX$ (do $(5)$) nên $\triangle S_1HK_1\sim \triangle ABX$ (góc - góc)

$\Rightarrow $ $\tfrac{S_1H}{HK_1}=\tfrac{AB}{BX}$ $\Rightarrow $ $\tfrac{S_1H}{HA}=\tfrac{AB}{BC}$ $\Rightarrow $ $\triangle ABC\sim \triangle S_1HA$ (cạnh - góc -cạnh) $\implies $ $S_1A\perp AC$. Tương tự $T_1A\perp AB$.

Từ đó theo tính chất hình bình hành $O$ là trung điểm $M_1N_1$. Gọi $P_1$ là hình chiếu của $A$ lên $M_1N_1$.

Ta có $\angle BPC=360^\circ-180^\circ+\angle A-180^\circ+\angle A=2\angle A=\angle BOC$ nên tứ giác $BP_1OC$ là tứ giác nội tiếp. $\qquad (6)$

Mặt khác do $\angle BPA=\angle CPA=180^\circ-\angle A$ nên $P_1A$ là phân giác $\angle BP_1C$ $\qquad (7)$

Từ $(6)$ và $(7)$ ta thu được $AP_1$ là đường đối trung của $\triangle ABC$ $\implies M_1N_1\parallel OQ$ $\qquad (8)$

Do $BM_1=HS_1=K_2S$ (do $(5)$) nên $K_2SM_1B$ là hình bình hành $\implies M$ là trung điểm $K_2M_1$.

Tương tự và theo tính chất đường trung bình thì $MN\parallel M_1N_1$ $\qquad (9)$

Từ $(8)$ và $(9)$ ta suy ra $MN\parallel OQ$. 




#663298 Tuần 5 tháng 11/2016 : Mở rộng bài toán hình học trường đông tại Vinh năm 2016

Đã gửi bởi baopbc on 28-11-2016 - 19:10 trong Chuyên mục Mỗi tuần một bài toán Hình học

Như vậy thầy Hùng đã đưa ra lời giải bài cũ trong tuần 5 tháng 11 và kèm theo đó là bài toán mới, xin trích dẫn lại bài toán mới,

 

Cho tam giác $ABC$ nội tiếp đường tròn $(O)$. $P$ nằm trên cạnh. Các  đường tròn $(PAB)$, $(PCA)$ lần lượt cắt $CA$, $AB$ tại $E$, $F$ khác $A$. $K$ là tâm ngoại tiếp của tam giác $AEF$. Tiếp tuyến tại $A$ của $(O)$ cắt $BC$ tại $Q$. Trên $QK$ lấy $S$, $T$ sao cho $ET\perp AC$, $FS\perp AB$. $M$, $N$ lần lượt là trung điểm của $CT$, $BS$.

Chứng minh rằng $MN\parallel OQ$.

Post 363.PNG

Hình vẽ bài toán




#655528 Tuần 4 tháng 9/2016: Đường vuông góc trên cấu hình đường tròn Mixlinear

Đã gửi bởi baopbc on 25-09-2016 - 19:00 trong Chuyên mục Mỗi tuần một bài toán Hình học

Như vậy thầy Hùng đã đưa ra lời giải bài cũ trong tuần 4 tháng 9 và kèm theo đó là bài toán mới, xin trích dẫn lại bài toán đó.

 

Cho tam giác $ABC$ nội tiếp trong đường tròn $(O)$ đường kính $AS$. Đường tròn $(K)$ tiếp xúc $CA,AB$ và tiếp xúc trong $(O)$ tại $D$. Trung trực $AD$ cắt tiếp tuyến tại $S$ của $(O)$ tại $T.P$ đối xứng với $D$ qua $TK$. Trung trực $AP$ cắt $PK$ tại $R.AK$ cắt $(O)$ tại $X$ khác $A.DX$ cắt $BC$ tại $G$. Lấy $Q$ trên trung trực $AX$ sao cho $AQ\perp BC$. Chứng minh rằng $QR\perp AG$.

Post 339.PNG

Hình vẽ bài toán

 




#655593 Tuần 4 tháng 9/2016: Đường vuông góc trên cấu hình đường tròn Mixlinear

Đã gửi bởi baopbc on 26-09-2016 - 13:39 trong Chuyên mục Mỗi tuần một bài toán Hình học

Lời giải của em.

Post 340.PNG

Hình 1

Tiếp tuyến tại $D$ của $\odot (K)$ cắt $BC$ tại $X$.

$SK$ cắt $\odot (O)$ tại $R$.

$\odot (K)$ tiếp xúc $CA,AB$ theo thứ tự tại $E$ và $F$.

Gọi $Z$ là giao điểm của $AD$ với $\odot (K)$.

Do $DZ,EA,FA$ đồng quy nên tứ giác $DFZE$ điều hòa suy ra tiếp tuyến tại $Z,D$ của $\odot (K)$ cắt nhau tại $Y$ thuộc $EF$.

Do đó $\mathcal{P}_{Y/\odot (O)}=YD^2=\overline{YF}\cdot \overline{YE}=\mathcal{P}_{Y/\odot (AEF)}$.

Từ đó $Y$ thuộc trục đẳng phương của $\odot (AEF)$ và $\odot (O)$.

Do $\angle ARK=90^\circ$ nên $R$ thuộc $\odot (AEF)$ suy ra $A,R,Y$ thẳng hàng.

Từ đó $(AR,AD,AF,AE)=-1$ nên tứ giác $RBDC$ điều hòa suy ra tiếp tuyến tại $R,D$ của $\odot (O)$ cắt nhau trên $BC$ hay $XR$ tiếp xúc $\odot (O)$.

Gọi $L$ là giao điểm khác $D$ của $DO$ với $\odot (O)$ thì $TL$ tiếp xúc $\odot (O)$.

Từ đó theo tính chất của tứ giác ngoại tiếp $DL,RS,XT$ đồng quy hay $XL$ đi qua $K$.

Do đó $P$ là đối xứng của $D$ qua $XK$ hay $\odot (BPC)$ tiếp xúc $\odot (K)$.

Post 341.PNG

Hình 2

Do $AQ,AO$ đẳng giác nên $Q$ là đối xứng của $O$ qua $AX$.

Từ đó đường tròn tâm $Q$ bán kính $QA$ là đối xứng của $\odot (O)$ qua $AX$. 

$D'$ là điểm đối xứng của $D$ qua $AX$

Gọi $M$ là giao điểm của $BE$ và $CF$ ;

$J$ là giao điểm của $GD$ với $\odot (K)$.

$G_1,G_2$ lần lượt là giao điểm của $\odot (K)$ với $BC$.

$L'$ là giao điểm của $AD'$ với $\odot (K)$.

Do tứ giác $PG_1DG_2$ điều hòa nên $(JD,JP,JG_1,JG_2)=-1$.

Từ đó gọi $H$ là giao điểm của $JP$ với $BC$ thì $(GHG_1G_2)=-1$.

Mặt khác do $DG_1,DG_2$ đẳng giác trong $\angle BDC$ nên $HG$ là phân giác ngoài $\angle G_1DG_2$ suy ra $DH$ là phân giác $\angle G_1DG_2$.

Do đó $DH$ đi qua $I$ là trung điểm $EF$.

Do $DA$ là đường đối trung của $\triangle EDF$ nên $DA,DI$ đẳng giác trong $\angle EDF$.

Mặt khác do $AD,AD'$ đối xứng nhau qua $AX$ nên $DH$ đi qua $L'$.

Áp dụng định lí $\text{Pascal's}$ cho hệ điểm $\begin{pmatrix} L' & P & D\\ J & D & P \end{pmatrix}$ ta suy ra $PL',DJ,BC$ đồng quy hay $PL'$ đi qua $G$.

Từ đó do tứ giác $L'ED'F$ điều hòa nên $(PL',PD',PE,PF)=-1 \Leftrightarrow (PG,PD',PE,PF)=-1$.

Do đó $PD',EF,AM$ đồng quy.  $(1)$

Gọi $N$ là giao điểm của hai tiếp tuyến tại $P$ và $D'$ của $\odot (K)$.

Xét cực đối cực đối với $\odot (K)$, do $AM,EF,PD'$ lần lượt là đường đối cực của $G,A,N$ đối với $\odot (K)$ nên theo $(1)$ thì $N,A,G$ thẳng hàng.

Mặt khác theo định lí về tâm đẳng phương thì tiếp tuyến tại $P,D'$ của $\odot (K)$ và trục đẳng phương của $\odot (Q)$ và $\odot (R,RA)$ đồng quy.

Từ đó$AG$ là trục đẳng phương của $\odot (Q)$ và $\odot (R,RA)$.

Do đó $AG\perp QR$. $\blacksquare$




#650715 Tuần 4 tháng 8/2016: Bài toán qua tâm

Đã gửi bởi baopbc on 21-08-2016 - 21:51 trong Chuyên mục Mỗi tuần một bài toán Hình học

Xin lỗi mọi người vì bài toán tuần này được mình đưa lên hơi muộn! :(

 

Lời giải bài toán cũ đã có tại tuần 4 tháng 8 và kèm theo đó là bài toán mới, xin được trích dẫn lại bài toán đó

 

Cho tam giác $ABC$ có trung tuyến $AM$. Lấy điểm $H$ sao cho $AH\perp BC$ và $HM\perp AM.P$ đối xứng $H$ qua $M.K,L$ là hình chiếu của $P$ lên cạnh $CA,AB$. Trên cạnh $CA,AB$ lấy $Q,R$ sao cho $AQ=2KC$ và $AR=2BL.AM$ cắt $QR$ tại $N$. Chứng minh rằng $PN$ đi qua tâm đường tròn ngoại tiếp tam giác $ABC$.

Post 294.PNG

Hình vẽ bài toán




#650753 Tuần 4 tháng 8/2016: Bài toán qua tâm

Đã gửi bởi baopbc on 22-08-2016 - 09:51 trong Chuyên mục Mỗi tuần một bài toán Hình học

Bài tuần này Hoàng giải vậy thì đẹp rồi, sau đây là lời giải của em! :)

Post 295.PNG

Hình vẽ bài toán

Lời giải bài toán. Gọi $I$ là trung điểm $AP$ thì $I$ là tâm đường tròn đường kính $AP$. Từ đó ta có $\angle MAP=\angle MAH=\angle PMC$ nên đường tròn $(AP)$ tiếp xúc $BC$.

Từ đó $BL.BA=CK.CA=CM^2\implies AR.AB=AQ.AC=2CM^2$ nên tứ giác $BRQC$ nội tiếp.

Gọi $S$ là giao điểm của $AM$ với đường tròn $(ABC)$. Ta có $\angle ASB=\angle ACB=\angle ARQ$ nên tứ giác $BRNS$ nội tiếp $\implies AN.AS=AR.AB=2CM^2$.

Mặt khác do tứ giác $ABSC$ nội tiếp nên $AM.MS=CM^2\implies AN.AS=2AM.MS\implies \frac{AM}{AN}=\frac{AS}{2MS}$

Gọi $T$ là điểm đối xứng của $A$ qua $O, F$ là giao điểm của $PO$ với $AH$ thì $AFTP$ là hình bình hành suy ra $AF=PT$. Gọi $X$ là giao điểm của $OM$ với $TS$. Do $MX\parallel PR\perp BC,MP\parallel XT\perp AM$ nên $MPTX$ là hình bình hành $\implies AF=PT=MX$.

Gọi $E$ là trung điểm $ST$ thì $OE\parallel MS$ nên theo định lí Thales thì $\frac{OM+AF}{AF}=\frac{OX}{MX}=\frac{OE}{MS}=\frac{AS}{2MS}=\frac{AM}{AN}$.

Từ đó theo định lí Thales đảo thì $O,N,F$ thẳng hàng hay $PN$ đi qua $O$. $\blacksquare$

 




#646279 Tuần 4 tháng 7/2016: Một năm "Mỗi tuần một bài toán"

Đã gửi bởi baopbc on 24-07-2016 - 18:32 trong Chuyên mục Mỗi tuần một bài toán Hình học

Trích lời dẫn của thầy Trần Quang Hùng trên blog "Hình học sơ cấp".

 


"Đôi lời chia sẻ cho hành trình một năm của "Mỗi tuần một bài toán"!

 

Như vậy là chuyên mục "Mỗi tuần một bài toán" bắt đầu từ tuần 1 tháng 8 năm 2015 cho tới hết tuần 4 tháng 7 năm 2016 đã tròn một năm. Chuyên mục đã giữ được một sự ổn định tương đối để có thể duy trì đều đặn hàng tuần trong một năm qua. Chuyên mục đã nhận được sự ủng hộ nhiệt tình từ rất nhiều bạn đọc trên cả nước và đặc biệt là được ban quản trị diễn đàn toán học hết sức tạo điều kiện hỗ trợ để có một box riêng tại đây. Đó thực sự là sự ủng hộ và đồng thuận rất lớn từ cộng đồng các bạn yêu toán trên diễn đàn toán học. Nhân dịp bài toán kết thúc một năm, mình xin được gửi lời cám ơn chân thành tới ban quan trị diễn đàn toán học cùng với tất cả các bạn đã tham gia giải bài, nhiệt tình ủng hộ chuyên mục và đặc biệt là các học trò của tôi là các bạn Phạm Quang ToànNguyễn Tiến Dũng và Nguyễn Đức Bảo đã luôn sát cánh bên tôi để chuyên mục có được một sự ổn định và đều đặn. Mình xin hứa sẽ làm cho chuyên mục tiếp diễn đều và sẽ luôn phát triển hơn nữa về chất lượng để cám ơn sự ủng hộ nhiệt tình của các bạn yêu toán và yêu hình học.

 

Tuần cuối, tháng 7 năm 2016.

Trần Quang Hùng"

 

Lời giải bài cũ đã được thầy Hùng đưa lên tại tuần 4 tháng 7 và kèm theo đó là bài toán mới.

Cho tam giác nhọn $ABC$ với đường cao $AD,BE,CF.M,N$ là trung điểm của $HC,HE$. Trên $EF$ lấy $P$ sao cho $MP\parallel DE.Q$ thuộc $BC$ sao cho $AQ\perp AB.AD$ cắt $EF$ tại $R$. Trên $BR$ lấy $S$ sao cho $ES\parallel NP$. Chứng minh rằng $QE,AB,SD$ đồng quy.

Post 269.PNG

 

 

hhdsdhaksdhkasd.




#646293 Tuần 4 tháng 7/2016: Một năm "Mỗi tuần một bài toán"

Đã gửi bởi baopbc on 24-07-2016 - 21:01 trong Chuyên mục Mỗi tuần một bài toán Hình học

Tổng quát bài toán cho điểm bất kì.

Tổng quát. Cho tam giác $ABC.H$ là một điểm bất kì trong tam giác. $\triangle DEF$ là tam giác $\textbf{Cevian}$ của $H$ đối với $\triangle ABC$. Đường thẳng qua $A\parallel CF$ cắt $BC$ tại $Q$. Gọi $M,N$ theo thứ tự là trung điểm $HC,HE$. Trên $EF$ lấy $P$ sao cho $MP\parallel DE$.Trên $BR$ lấy $S$ sao cho $ES\parallel PN$. Chứng minh rằng $DS,QE,AB$ đồng quy.

Post 270.PNG

PS. Đến đây thì ý tưởng giải có lẽ khá rõ! :) Đại số hóa hoặc dùng hàng điểm.




#641385 Tuần 4 tháng 6 năm 2016: Đường thẳng đi qua điểm cố định

Đã gửi bởi baopbc on 20-06-2016 - 12:52 trong Chuyên mục Mỗi tuần một bài toán Hình học

Như vậy bài Tuần 3 tháng 6 đã được thầy Hùng cho lời giải tại đây kèm theo đó là bài toán mới. Xin trích dẫn lại bài toán mới:

 

Cho tam giác $ABC$ nội tiếp đường tròn $(O)$ cố định với $B,C$ cố định và $A$ di chuyển trên $(O)$. Gọi $P,Q$ là hai điểm Isodynamic của tam giác $ABC$. $K$ đối xứng $A$ qua $BC$. $OK$ cắt đường tròn ngoại tiếp tam giác $KPQ$ tại $R$ khác $K$. Chứng minh rằng đường thẳng $AR$ luôn đi qua điểm cố định khi $A$ thay đổi.

Lời giải của em không dùng nghịch đảo và có lẽ cũng đơn giản hơn! :)

Lời giải. Hai điểm isodynamic là giao điểm của 3 đường tròn Apollonius của tam giác $ABC$ dựng trên $BC,CA,AB$ lần lượt với tỉ số $\frac{AB}{AC},\frac{BC}{BC},\frac{CA}{CB}$. Do tâm đường tròn ngoại tiếp tam giác $APQ$ nằm trên $BC$ nên $K$ thuộc $(APQ)$.

Post 218.png

Giả sử $(O_a)$ là đường tròn Apollonius ứng với $BC$ theo tỉ số $\frac{AB}{AC}$

$N\equiv (O)\cap (APQ)$. Do đường tròn Apollonius trực giao với $(O)$ nên $OA,ON$ lần lượt là các tiếp tuyến tại $A,N$ của $(O_a)$

$\implies ARNK$ là tứ giác điều hòa $\implies A(ORNK)=-1$

$T\equiv AN\cap $ đường thẳng qua $O$ vuông góc với $BC\implies AR$ chia đôi $OT$. Mặt khác do $ABRC$ là tứ giác điều hòa nên tiếp tuyến tại $B,C$ và $AN$ đồng quy $\implies T$ cố định hay $AR$ đi qua một điểm cố định khi $A$ thay đổi.$\blacksquare$




#631102 TURKEY Team Selection Test 2016

Đã gửi bởi baopbc on 03-05-2016 - 22:59 trong Thi HSG Quốc gia và Quốc tế

Bài 6. Cho tam giác $ABC$ cân tại $A$ với $D$ là trung điểm của $BC$. Một đường thẳng qua $D$ cắt $AB$ tại $K$, $AC$ tại $L$. Điểm $E$ trên cạnh $BC$ ($E \neq D$), $P$ trên $AE$ sao cho $\angle KPL = 90^{\circ} - \frac{1}{2}\angle KAL$; $E$ nằm giữa $A$ và $P$. Đường tròn ngoại tiếp tam giác $PDE$ cắt $PK$ tại $X$ ($X \neq P$) và cắt $PL$ tại $Y$ ($Y \neq L$). Đường $DX$ cắt $AB$ tại $M$, đường $DY$ cắt $AC$ tại $N$. Chứng minh rằng bốn điểm $P, M, A, N$ cùng thuộc một đường tròn.

Bài số $6$ đúng với vị trí trong đề của nó! :)

Lời giải. Gọi $T$ là giao của đường thẳng qua $K$ song song với $BC$ với $AC$. Do $\angle KPL=90^\circ-\frac{1}{2}\angle KAL$ nên $K,T,P,L$ đồng viên.

Quy ước $\odot (KTPL)=\omega .TD$ cắt $AB$ tại $H$. Dễ thấy $KTLH$ là hình thang cân nên $H$ thuộc $\omega $.

Post 100.png

Hình vẽ $1$

Ta có: $\angle MHP=180^\circ=\angle KLP$. Mặt khác theo kết quả cũ thì $\odot (DEP)$ tiếp xúc $\omega $ nên $XY\parallel KL\Rightarrow \angle XDP=\angle XYP=180^\circ-\angle MHP$ nên $D,M,H,P$ đồng viên $\Rightarrow \angle DPM=\angle DHM=\angle KPT\Rightarrow \angle XPM=\angle DPT\Rightarrow \triangle KPM\sim \triangle TPD(g.g)\Rightarrow \frac {TD}{KM}=\frac {TP}{KP}\Rightarrow \frac {KD}{MK}=\frac {TP}{KP}. (1)$

Ta chứng minh $\frac {DX}{XM}=\frac {EC}{BE}\Leftrightarrow \frac {sin \angle XKD.KD}{sin \angle XKM.KM}=\frac {sin \angle EAC}{sin \angle EAB}$ (Luôn đúng theo $(1)$. 

Gọi $S,T$ lần lượt là giao của $ND$ với $PX,MD$ với $PY$.

Post 101.png

Hình vẽ $2$

Dễ thấy $\triangle SDX\sim \triangle ACE(g.g)$ nên $A,N,P,S$ đồng viên. Tương tự ta suy ra $A,M,P,T$ đồng viên. Mặt khác sử dụng kết quả trên ta suy ra $\triangle SDM\sim \triangle ACB\Rightarrow SD=SM$. Tương tự thì $TD=TN$.

$\angle DNC=\angle EPX=\angle BDM$ nên $D$ là tâm nội tiếp tam giác $AMN\Rightarrow S$ thuộc $\odot (ANM).\Rightarrow P$ thuộc $\odot (AMN).\blacksquare$.

P.s: Bài này còn rất nhiều kết quả đẹp khác! :)

Thầy Hùng có đưa lên AopS một bài toán nghịch đảo bài này và bất ngờ là nó được giải rất đơn giản! :) Mọi người xem lời giải ở đây.




#630695 TURKEY Team Selection Test 2016

Đã gửi bởi baopbc on 02-05-2016 - 07:01 trong Thi HSG Quốc gia và Quốc tế

Có tí thắc mắc, đoạn này là bổ đề nào vậy nhỉ?

Bổ đề này khá cơ bản trong tam giác. 

Phần thuận hiển nhiên đúng.

Phần đảo: $X$ là giao của $DE$ với $BC$. Khi đó theo định lí $Brokard$ thì $OX\perp AH$. Mặt khá $AH\perp BC$ nên $K$ phải thuộc $BC\Rightarrow H$ là trực tâm.




#630513 TURKEY Team Selection Test 2016

Đã gửi bởi baopbc on 01-05-2016 - 12:05 trong Thi HSG Quốc gia và Quốc tế

 

 

Bài 1. Cho tam giác $ABC$ nhọn, $P$ thuộc đường cao hạ từ $A$. Các đường thẳng $BP, CP$ lần lượt cắt các cạnh $CA, AB$ tương ứng tại $D, E$. Tiếp tuyến kẻ từ $D$ và $E$ tới $(BPC)$ tiếp xúc tại $K, L$ tương ứng ($K, L$ nằm trong tam giác). Đường thẳng $KD$ giao với $(AKC)$ tại $M$ ($M \neq K$) và đường $LE$ giao với $(ALB)$ tại $N$ ($N \neq L$). Chứng minh rằng $\frac{DK}{DM} = \frac{EL}{EN}$ khi và chỉ khi $P$ là trực tâm của tam giác $ABC$.

Lời giải. $AC,AB$ lần lượt cắt $\odot (BPC)$ tại $P,Q$.

Post 93.png

Hình vẽ bài toán

Ta có: $\angle BQL=\angle BCL=\angle BLE=\angle BAN\Rightarrow AN\parallel QL\Rightarrow \frac {AE}{EQ}=\frac {NE}{NL}$

Tương tự ta suy ra $\frac {DM}{DK}=\frac {DA}{DP}$

$\Rightarrow \frac {DK}{DM}=\frac {EL}{EN}$ khi và chỉ khi $DE\parallel PQ$.

Mặt khác do $B,C,Q,L$ đồng viên nên $\angle PQB=\angle BCA$

$\Rightarrow DE\parallel PQ$ khi và chỉ khi $BCDE$ là tứ giác nội tiếp hay $P$ là trực tâm tam giác $ABC$.

Nhận xét: Nên phát biểu là $DK=DM$ và $EL=EN$ thì bài sẽ đẹp hơn! :)